0

箙と余代数

207
0
$$\newcommand{C}[0]{\mathcal{C}} \newcommand{catsemigrpzero}[0]{\mathrm{SemiGrp}^0} \newcommand{id}[0]{\mathrm{id}} \newcommand{idealgen}[0]{\mathord{\uparrow}} \newcommand{Q}[0]{\mathcal{Q}} \newcommand{ReesJ}[0]{\mathrel{\mathcal{J}}} \newcommand{setex}[1]{\left\{#1\right\}} \newcommand{setin}[2]{\left\{#1\mathrel{}\middle|\mathrel{}#2\right\}} \newcommand{supp}[0]{\operatorname{\mathrm{supp}}} \newcommand{tip}[0]{\operatorname{\mathrm{tip}}} \newcommand{W}[0]{\mathcal{W}} $$

圏の中の箙

箙の定義

$\Q$を小圏$1\overset{s}{\underset{t}{\rightrightarrows}}0$とする($\id$は省略している).圏$\C$に対し,関手$\Gamma\colon\Q\to\C$$\C$内の箙という.つまり,$\C$内の箙$\Gamma$とは,$\C$の対象$\Gamma_0,\Gamma_1$とその間の射$s,t\colon\Gamma_1\to\Gamma_0$の組$(\Gamma_0,\Gamma_1,s,t)$のことである.
$\C$内の箙の圏を関手圏$[\Q,\C]$で定める.従って,$\C$内の箙の間の射とは自然変換$\theta\colon\Gamma\Rightarrow\Gamma'$,すなわち射$\theta_0\colon\Gamma_0\to\Gamma'_0$, $\theta_1\colon\Gamma_1\to\Gamma'_1$の組であって,下図を可換にするものである:
\begin{xy}\xymatrix{ \Gamma_1 \ar[r]^{\theta_1} \ar[d]_s & \Gamma'_1 \ar[d]^s & \Gamma_1 \ar[r]^{\theta_1} \ar[d]_t & \Gamma'_1 \ar[d]^t\\ \Gamma_0 \ar[r]_{\theta_0} & \Gamma'_0 & \Gamma_0 \ar[r]_{\theta_0} & \Gamma'_0.\\ }\end{xy}

箙の拡張

$\mathbb{N}$を非負整数の集合とする.$W=\langle s,t\rangle\cong\mathbb{N}^2$を自由可換モノイドとし,各$n\in \mathbb{N}$に対し$W_n$$W$の長さ$n$の要素全体からなる部分集合とする.小圏$\W$を次のように定める.$\W$の対象は$\mathbb{N}$であり,射の集合は
\begin{align} \W(n,m)=\begin{cases} W_{n-m} &\text{if }n\geq m\\ \emptyset &\text{if otherwise} \end{cases} \end{align}
である.射の結合は$W$の積で定める.$\Q$$\W$の充満部分圏である.
$\C$をpullbackを持つ圏とする.このとき$\C$の箙$\Gamma\colon\Q\to\C$は関手$\W\to\C$に拡張される.

$\C$をpullbackを持つ圏とする.このとき任意の$\Gamma\in[\Q,\C]$に対し,包含関手$\iota\colon\Q\to\W$に沿った$\Gamma$の各点右Kan拡張$\langle\bar{\Gamma},\id\rangle$が存在する.とくに,$\bar{(\cdot)}\colon[\Q,\C]\to[\W,\C]$は,$-\circ\iota\colon[\W,\C]\to[\Q,\C]$の右随伴であるような関手を定める.
\begin{xy}\xymatrix{ \W \ar[rrd]^{\bar{\Gamma}}\rtwocell<\omit>{<5>\id}&&\\ \Q \ar@{^{(}->}[u]^\iota \ar[rr]_\Gamma & & \C\\ }\end{xy}

$n\in\W$と定値関手$n\colon\ast\to\W$とみなし,コンマ圏$n/\iota$を考える.自然な忘却関手と$\Gamma$の合成を
$$F_n\colon n/\iota\to\Q\xrightarrow[\Gamma]{}\C$$
とする.$0/\iota\cong\ast$だから,$\bar{\Gamma}_0:=\lim F_0=\Gamma_0$である.
$n\geq1$とする.コンマ圏$n/\iota$の対象は$\W$における射$n\to1$, $n\to0$, すなわち$W_{n-1}\cup W_n$である.$n/\iota$における射は,$\Q$$\id$から来るものを除けば
\begin{xy}\xymatrix{ n \ar[r]^w \ar[rd]_{ws} & 1 \ar[d]^s & n \ar[r]^w \ar[rd]_{wt} & 1 \ar[d]^t\\ & 0 && 0\\ }\end{xy}
($w\in W_{n-1}$), である.自然な忘却関手$n/\iota\to\Q$によって対象は
\begin{align} w\mapsto \begin{cases} 1 &\text{if }w\in W_{n-1},\\ 0 &\text{if otherwise} \end{cases} \end{align}
に写り,射は$(w\to ws)\mapsto s$, $(w\to t)\mapsto t$に写る.従って,図式$F_n$の極限とは,対象$\bar{\Gamma}_n\in\C$と射$f_w\colon\bar{\Gamma}_n\to\Gamma_1$, ($w\in W_{n-1}$), $g_{w'}\colon\bar{\Gamma}_n\to\Gamma_0$, ($w'\in W_n$)の組で,
\begin{xy} \begin{gathered}\xymatrix@=12pt{ & \bar{\Gamma}_n \ar[lddd]_{g_{n,0}} \ar[dd]^{f_{n-1,0}}\\\\ & \Gamma_1 \ar[ld]^s\\ \Gamma_0\\ }\end{gathered}\quad \begin{gathered}\xymatrix@=12pt{ & \bar{\Gamma}_n \ar[ldd]_{f_{n-i,i-1}} \ar[ddd]_{g_{n-i,i}} \ar[rdd]^{f_{n-i-1,i}} &\\\\ \Gamma_1 \ar[rd]_t && \Gamma_1 \ar[ld]^s\\ & \Gamma_0 &\\ }\end{gathered}\quad \begin{gathered}\xymatrix@=12pt{ \bar{\Gamma}_n \ar[dd]_{f_{0,n-1}} \ar[rddd]^{g_{0,n}} &\\\\ \Gamma_1 \ar[rd]_t &\\ & \Gamma_0\\ }\end{gathered} \end{xy}
($i=1,\dots,n-1$), を可換にするもので普遍的なものである.ただし$f_{s^it^j}$$f_{i,j}$などと略記している.$f_{n-1,0},\dots,f_{0,n-1}$が与えられれば$g_{n,0},\dots,g_{0,n}$は自動的に定まることに注意して条件を書き直せば次のようになる: 対象$\bar{\Gamma}_n\in\C$と射$f_{n-1,0},\dots,f_{0,n-1}\colon\bar{\Gamma}_n\to\Gamma_1$で,
\begin{xy}\xymatrix@C=5pt@R=12pt{ &&&& \bar{\Gamma}_n \ar[llldd]_{f_{n-1,0}} \ar[ldd]_{f_{n-2,1}} \ar@{}[dd]|{\cdots} \ar[ddrr]^{f_{0,n-1}}&&&\\\\ & \Gamma_1 \ar[ld]^s \ar[rd]_t && \Gamma_1 \ar[ld]^s \ar[rd]_t & \quad\cdots && \Gamma_1 \ar[ld]_s \ar[rd]^t &\\ \Gamma_0 && \Gamma_0 && \Gamma_0 & \cdots \Gamma_0&&\Gamma_0\\ }\end{xy}
を可換にするもので普遍的なものである.この極限は,$\C$がpullbackを持つことから存在する.実際,$\bar{\Gamma}_1=\Gamma_1$であり,$n\geq2$については逐次pullbackをとればよい:
\begin{xy}\xymatrix@C=5pt@R=12pt{ &&&&& \bar{\Gamma}_n \ar@{}[dd]|{\mathrm{pb.}} \ar[lld] \ar[rrdd] &&\\ &&& \bar{\Gamma}_{n-1} \ar[lld] \ar@{}[d]|{\cdots} \ar[rrd] &&&\\ & \Gamma_1 \ar[ld] \ar[rd] &&&& \Gamma_1 \ar[ld] \ar[rd] && \Gamma_1 \ar[ld] \ar[rd] &\\ \Gamma_0 && \Gamma_0 & \cdots & \Gamma_0 && \Gamma_0 && \Gamma_0.\\ }\end{xy}

$\bar{\Gamma}_w$の計算

$\Gamma\in[\Q,\C]$とする.$\bar{\Gamma}\in[\W,\C]$であるから,$\W$の射$w\colon n\to m$に対し,$\C$の射$\bar{\Gamma}_w\colon \bar{\Gamma}_n\to\bar{\Gamma}_m$が得られるが,この射も同じ記号$w$で表す.

任意の$i,j\geq0$に対し,下図はpullbackを与える:
\begin{xy}\vcenter{\xymatrix{ \bar{\Gamma}_{i+j} \ar[r]^{t^i} \ar[d]_{s^j} & \bar{\Gamma}_j \ar[d]^{s^j}\\ \bar{\Gamma}_i \ar[r]_{t^i} & \bar{\Gamma}_0.\\ }}\tag{$\square$}\label{shikaku}\end{xy}

$n=i+j$とおく.$W$は可換ゆえ\eqref{shikaku}は可換である.別の可換な四角形
\begin{xy}\xymatrix{ X \ar[r]^{f} \ar[d]_{g} & \bar{\Gamma}_j \ar[d]^{s^j}\\ \bar{\Gamma}_i \ar[r]_{t^i} & \bar{\Gamma}_0\\ }\end{xy}
が得られたとする.このとき下図は可換である:
\begin{xy}\xymatrix@C=5pt@R=12pt{ &&&& X \ar[lld]_g \ar[rrd]^f &&&&\\ && \bar{\Gamma}_i \ar[ld]_{s^{i-1}} \ar@{}[dd]|{\cdots} \ar[rd]^{t^{i-1}}&&&& \bar{\Gamma}_j \ar[ld]_{s^{j-1}} \ar@{}[dd]|{\cdots} \ar[rd]^{t^{j-1}}&&\\ & \Gamma_1 \ar[ld]^s \ar[rd]_t && \Gamma_1 \ar[rd]_t && \Gamma_1 \ar[ld]^s \ar[rd]_t && \Gamma_1 \ar[rd]_t &\\ \Gamma_0 && \Gamma_0\cdots && \Gamma_0 && \Gamma_0\cdots && \Gamma_0\\ }\end{xy}
よって$\bar{\Gamma}_n$の普遍性から仲介射$\varphi\colon X\to\bar{\Gamma}_n$が一意にのびて,
\begin{align} s^{n-k}t^{k-1}\varphi&=s^{i-k}t^{k-1}g &(k=1,\dots,i),\\ s^{j-l}t^{i+l-1}\varphi&=s^{j-l}t^{l-1}f &(l=1,\dots,j) \end{align}
が成り立つ.
\begin{xy}\xymatrix@C=5pt@R=12pt{ &&& X \ar[d]^\varphi &&&\\ &&& \bar{\Gamma}_n \ar[lld]_{s^{n-1}} \ar@{}[dd]|{\cdots} \ar[rrd]^{t^{n-1}} &&&\\ & \Gamma_1 \ar[ld]^s \ar[rd]_t &&&& \Gamma_1 \ar[ld]^s \ar[rd]_t &\\ \Gamma_0 && \Gamma_0 & \cdots & \Gamma_0 && \Gamma_0\\ }\end{xy}
とくに$k=1,\dots,i$に対し$s^{i-k}t^{k-1}\cdot s^j\varphi=s^{i-k}t^{k-1}\cdot g$であるから,$\bar{\Gamma}_i$の普遍性から$g=s^j\varphi$である.同様に$f=t^i\varphi$もわかる.以上により\eqref{shikaku}がpullbackと分かった.

$\bar{\theta}$の計算

$\bar{(\cdot)}\colon[\Q,\C]\to[\W,\C]$は関手であるから,射$\theta\colon\Gamma\Rightarrow\Gamma'$に対して射$\bar{\theta}\colon\bar{\Gamma}\Rightarrow\bar{\Gamma}'$が定まる.この$\bar{\theta}$の成分$\bar{\theta}_n\colon\bar{\Gamma}_n\to\bar{\Gamma}'_n$の計算方法を述べる.
\begin{xy}\xymatrix{ \bar{\Gamma}_{i+j} \ar[r]^{t^i} \ar[d]_{s^j} & \bar{\Gamma}_j \ar[r]^{\bar{\theta}_j} \ar[d]^{s^j} & \bar{\Gamma}'_j \ar[dd]^{s^j}\\ \bar{\Gamma}_i \ar[r]_{t^i} \ar[d]_{\bar{\theta}_i} & \bar{\Gamma}_0 \ar[rd]^{\bar{\theta}_0} &\\ \bar{\Gamma}'_i \ar[rr]_{t^i} && \bar{\Gamma}'_0\\ }\end{xy}
\begin{align} s^j\bar{\theta}_jt^i &=\bar{\theta}_0s^jt^i=\bar{\theta}_0t^is^j\\ &=t^i\bar{\theta}_is^j \end{align}
であるから,$\bar{\Gamma}'_{i+j}$に関するpullbackの普遍性から仲介射$\bar{\Gamma}_{i+j}\to\bar{\Gamma}'_{i+j}$が一意にのびるが,これが$\bar{\theta}_{i+j}$である.従って$\theta\colon\Gamma\Rightarrow\Gamma'$が与えられれば,$\theta_0,\theta_1$から逐次$\bar{\theta}_n$が計算できる.

関手圏$[\W,\C]$と余代数

$\C$は(二項)積$\times$と終対象$I$を持つ圏とする.$\C$は自然にモノイダル構造を持つ.
$\bar{\Gamma}\in[\W,\C]$とする.任意の$i,j\geq0$に対し,$\C$の射
$$d_{i,j}:=\langle s^j,t^i\rangle\colon\bar{\Gamma}_{i+j}\to\bar{\Gamma}_i\times\bar{\Gamma}_j$$
を定める.
\begin{align}\xymatrix@C=12pt{ & \bar{\Gamma}_{i+j} \ar@/_/[ld]_{s^j} \ar[d]_{d_{i,j}} \ar@/^/[rd]^{t^i} &\\ \bar{\Gamma}_i & \bar{\Gamma}_i\times\bar{\Gamma}_j \ar[l] \ar[r] & \bar{\Gamma}_j\\ }\end{align}

$k$を体とし,$\mathrm{Vec}_k$$k$線形空間の圏で,テンソル積によるモノイダル構造を考える.$F\colon\C\to\mathrm{Vec}_k$を強モノイダル関手とする.$n\geq0$に対し$V_n=F(\bar{\Gamma}_n)$とおき,$V=\hat{F}(\bar{\Gamma}):=\bigoplus_{n\geq0}V_n$とおく.$\Delta\colon V\to V\otimes V$, $\varepsilon\colon V\to k$をそれぞれ,$v\in V_n$に対し
$$\Delta(v):=\sum_{i+j=n}\Delta_{i,j}(v),$$
\begin{align} \varepsilon(v):=\begin{cases} F_!(v) &\text{if }v\in V_0,\\ 0 &\text{if otherwise} \end{cases} \end{align}
により定める.ここで,
$$\Delta_{i,j}=F_{d_{i,j}}\colon V_n\to F(\bar{\Gamma}_i\times\bar{\Gamma}_j)\cong V_i\otimes V_j$$
であり,$!=!_{\bar{\Gamma}_0}\colon \bar{\Gamma}_0\to I$は唯一の射である.

$\hat{F}(\bar{\Gamma})$$k$上の(次数付き)余代数であり,また$\hat{F}$が関手$[\W,\C]\to\mathrm{Coalg}_k$を定めることを示す.

余単位律

$\varepsilon$は余単位的である.

$(!\times\id_{\bar{\Gamma}_n})\circ d_{0,n}=\langle!,\id_{\bar{\Gamma}_n}\rangle$であるから,$v\in V_n$に対し
$$(\varepsilon\otimes\id_V)\circ\Delta(v)=(\varepsilon\otimes\id_{V_n})\circ\Delta_{0,n}(v)=1\otimes v$$
である.$(\id\otimes\varepsilon)\circ\Delta$についても同様である.
\begin{xy} \begin{gathered}\xymatrix@C=12pt{ & \bar{\Gamma}_n \ar@/_/[ld]_{s^n} \ar[d]_{d_{0,n}} \ar@/^/[rd]^\id &\\ \bar{\Gamma}_0 \ar[d]_{!} & \bar{\Gamma}_0\times\bar{\Gamma}_n \ar[l] \ar[d]_{!\times\id} \ar[r] & \bar{\Gamma}_n \ar[d]^\id\\ I & I\times\bar{\Gamma}_n \ar[l] \ar[r] & \bar{\Gamma}_n\\ }\end{gathered} \overset{F}{\rightsquigarrow} \begin{gathered}\xymatrix@C=12pt{ V_n \ar[d]_{\Delta_{0,n}} \ar@/^12pt/[rdd]^\id &\\ V_0\otimes V_n \ar[d]_{\varepsilon\otimes\id} &\\ k\otimes V_n \ar[r]_\cong & V_n\\ }\end{gathered} \end{xy}

余結合律

$\C$の標準的な結合子を$a_{X,Y,Z}\colon(X\times Y)\times Z\to X\times(Y\times Z)$とする.下の補題では$a_{\bar{\Gamma}_i,\bar{\Gamma}_j,\bar{\Gamma}_k}=a_{i,j,k}$と略記している.

任意の$i,j,k\geq0$に対し,$a_{i,j,k}\circ(d_{i,j}\times\id_{\bar{\Gamma}_k})\circ d_{i+j,k}=(\id_{\bar{\Gamma}_i}\times d_{j,k})\circ d_{i,j+k}$である.

\begin{xy} \begin{gathered}\xymatrix@R=12pt@C=10pt{ &&& \bar{\Gamma}_{i+j+k} \ar[ldd]_{s^k} \ar[dd]_{d_{i+j,k}} \ar[rrddddd]^{t^{i+j}} &&&\\\\ && \bar{\Gamma}_{i+j} \ar[lddd]_{s^j} \ar[ddd]_{d_{i,j}} \ar[rddd]^{t^i} & \bar{\Gamma}_{i+j}\times\bar{\Gamma}_k \ar[l] \ar[dd]_{d_{i,j}\times\id} \ar[rrddd] &&&\\\\ &&& (\bar{\Gamma}_i\times\bar{\Gamma}_j)\times\bar{\Gamma}_k \ar[ld] \ar[rd] &&&\\ & \bar{\Gamma}_i & \bar{\Gamma}_i\times\bar{\Gamma}_j \ar[l] \ar[r] & \bar{\Gamma}_j && \bar{\Gamma}_k &\\ }\end{gathered} \begin{gathered}\xymatrix@R=12pt@C=10pt{ &&& \bar{\Gamma}_{i+j+k} \ar[llddddd]_{s^{j+k}} \ar[dd]^{d_{i,j+k}} \ar[rdd]^{t^{i}} &&&\\\\ &&& \bar{\Gamma}_i\times\bar{\Gamma}_{j+k} \ar[llddd] \ar[dd]^{\id\times d_{j,k}} \ar[r] & \bar{\Gamma}_{j+k} \ar[lddd]_{s^k} \ar[ddd]_{d_{j,k}} \ar[rddd]^{t^j} &&\\\\ &&& \bar{\Gamma}_i\times(\bar{\Gamma}_j\times\bar{\Gamma}_k) \ar[ld] \ar[rd] &&&\\ & \bar{\Gamma}_i && \bar{\Gamma}_j & \bar{\Gamma}_j\times\bar{\Gamma}_k \ar[l] \ar[r] & \bar{\Gamma}_k &\\ }\end{gathered} \end{xy}

図式を丁寧に見れば
\begin{eqnarray} \bar{\Gamma}_{i+j+k}\xrightarrow{(d_{i,j}\times\id)\circ d_{i+j,k}}(\bar{\Gamma}_i\times\bar{\Gamma}_j)\times\bar{\Gamma}_k\to&\bar{\Gamma}_i&=s^js^k=s^{j+k},\\ &\bar{\Gamma}_j&=t^is^k,\\ &\bar{\Gamma}_k&=t^{i+j} \end{eqnarray}
が分かる(直積から延びる射は射影(の合成)である).同様に
\begin{eqnarray} \bar{\Gamma}_{i+j+k}\xrightarrow{(\id\times d_{j,k})\circ d_{i,j+k}}\bar{\Gamma}_i\times(\bar{\Gamma}_j\times\bar{\Gamma}_k)\to&\bar{\Gamma}_i&=s^{j+k},\\ &\bar{\Gamma}_j&=s^kt^i=t^i s^k,\\ &\bar{\Gamma}_k&=t^jt^i=t^{i+j} \end{eqnarray}
である.従って題意が成り立つ.

$\Delta$は余結合的である.

$v\in V_n$に対し,補題により
\begin{align} (\Delta\otimes\id)(\Delta(v)) &=\sum_{i+j+k=n}(\Delta_{i,j}\otimes\id)(\Delta_{i+j,k}(v))\\ &=\sum_{i+j+k=n}(\id\otimes\Delta_{j,k})(\Delta_{i,j+k}(v))\\ &=(\id\otimes\Delta)(\Delta(v)) \end{align}
を得る.

射の構成

$\bar{\theta}\colon\bar{\Gamma}\rightarrow\bar{\Gamma}'$$[\W,\C]$の射とする.

任意の$i,j\geq0$に対し,$(\bar{\theta}_i\times\bar{\theta}_j)\circ d_{i,j}=d'_{i,j}\circ\bar{\theta}_{i+j}$である.
\begin{xy}\xymatrix{ \bar{\Gamma}_{i+j} \ar[r]^{d_{i,j}} \ar[d]_{\bar{\theta}_{i+j}} & \bar{\Gamma}_i\times\bar{\Gamma}_j \ar[d]^{\bar{\theta}_i\times\bar{\theta}_j}\\ \bar{\Gamma}'_{i+j} \ar[r]_{d'_{i,j}} & \bar{\Gamma}'_i\times\bar{\Gamma}'_j.\\ }\end{xy}

直積の普遍性から$(\bar{\theta}_i\times\bar{\theta}_j)\circ d_{i,j}=\langle\bar{\theta}_is^j,\bar{\theta}_jt^i\rangle=d'_{i,j}\circ\bar{\theta}_{i+j}$を得る.
\begin{xy}\xymatrix{ & \bar{\Gamma}_{i+j} \ar@/_15pt/[ldd]_{\bar{\theta}_is^j} \ar[d]_{\bar{\theta}_{i+j}} \ar@/^15pt/[rdd]^{\bar{\theta}_jt^i} &\\ & \bar{\Gamma}'_{i+j} \ar@/_/[ld]^{s^j} \ar[d]^{d'_{i,j}} \ar@/^/[rd]_{t^i} &\\ \bar{\Gamma}'_i & \bar{\Gamma}'_i\times\bar{\Gamma}'_j \ar[l] \ar[r] & \bar{\Gamma}'_j\\ }\end{xy}

$\hat{F}_\bar{\theta}\colon \hat{F}(\Gamma)\to\hat{F}(\Gamma')$を,$v\in V_n$に対し$\hat{F}_\bar{\theta}(v):=F_{\bar{\theta}_n}(v)\in V'_n$で定める.

$\hat{F}_\theta$は余代数準同型である.

$v\in V_n$とする.補題により,
\begin{align} (\hat{F}_\bar{\theta}\otimes\hat{F}_\bar{\theta})(\Delta(v)) &=(\hat{F}_\bar{\theta}\otimes\hat{F}_\bar{\theta})\left(\sum_{i+j=n}\Delta_{i,j}(v)\right)\\ &=\sum_{i+j=n}(F_{\bar{\theta}_i}\otimes F_{\bar{\theta}_j})(\Delta_{i,j}(v))\\ &=\sum_{i+j=n}\Delta'_{i,j}(F_{\bar{\theta}_n}(v))\\ &=\Delta(\hat{F}_\bar{\theta}(v)) \end{align}
を得る.$\varepsilon'\circ\hat{F}_\bar{\theta}=\varepsilon$は明らかである.

定義から,$\hat{F}_{\id_\bar{\Gamma}}=\id_{\hat{F}(\bar{\Gamma})}$, $\hat{F}_{\bar{\theta}\circ\bar{\theta}'}=\hat{F}_\bar{\theta}\circ\hat{F}_{\bar{\theta}'}$は明らかである.以上により,$\hat{F}\colon[\W,\C]\to\mathrm{Coalg}_k$が関手を定めることが分かった.

有限完備圏の箙と余代数

$\C$を有限完備圏とする.とくに$\C$はpullbackを持つから,初めの節より右Kan拡張$\mathrm{Ran}_\iota=\bar{(\cdot)}\colon[\Q,\C]\to[\W,\C]$が存在する.これと前節の関手$\hat{F}\colon[\W,\C]\to\mathrm{Coalg}_k$を合成することで$\C$の箙の圏から余代数の圏への関手が得られる.定理としてまとめておく.

有限完備圏$\C$と強モノイダル関手$F\colon\C\to\mathrm{Vec}_k$に対し,
$$\hat{F}\circ\mathrm{Ran}_\iota\colon[\Q,\C]\to\mathrm{Coalg}_k$$
は関手を定める.

$\mathrm{Set}$内の箙は単に箙という.箙$\Gamma$に対し,$\bar{\Gamma}_n$は長さ$n$の道全体の集合(と同型)である.$F$として自由ベクトル空間をとる関手を考えるとき,得られる余代数はpath coalgebraと呼ばれる.

memo

$M$: cancellative(?) abelian(?) monoid with a certain finiteness property like $|\setin{(i,j)}{i+j=n}|<\infty$
$\mathcal{M}$:
[objects] $M$
[morphisms] for $m,n\in M$,
$$\mathcal{M}(m,n)=\setin{(x,y)\in M\oplus M}{xny=m}$$
[composition] for $(x,y)\in \mathcal{M}(l,m)$, $(z,w)\in\mathcal{M}(m,n)$,
$$(z,w)\circ(x,y):=(xz,wy)$$
$\Gamma\in[\mathcal{M},\C]$$\rightsquigarrow$$\hat{F}(\Gamma)$ $M$-graded?

投稿日:2023921

この記事を高評価した人

高評価したユーザはいません

この記事に送られたバッジ

バッジはありません。

投稿者

コメント

他の人のコメント

コメントはありません。
読み込み中...
読み込み中